- Fri Apr 29, 2016 11:37 am
#23615
Complete Question Explanation
Must Be True—PR. The correct answer choice is (B)
Although this philosopher's assertion is stated in a fairly sophisticated manner, it can be restated in simpler terms:
We shouldn't be opposed to revealing truths we'd rather not acknowledge, or those
that might have negative influence on society. The question stem asks for the answer choice which most closely conforms to this basic principle.
Answer choice (A): Since this choice deals with illegally obtained information, it does not reflect the principle from the stimulus, which deals only with the assertion that we shouldn't be opposed to revealing certain truths.
Answer choice (B): This is the correct answer choice. This choice applies the principle from the stimulus to a medical context. We should not restrict such research even if negative consequences might result.
Answer choice (C): The philosopher makes no such specific exceptions to the general rule that we should not be opposed to unearthing the referenced truths.
Answer choice (D): This answer choice is incorrect for basically the same reason as incorrect answer choice (A) above. Since this choice deals with illegal means of obtaining information, the philosopher's assertion is not applicable in this context.
Answer choice (E): While the stimulus' argument is that we should not be opposed unearthing truths, this choice asserts that in certain contexts we should not have to adhere too closely to the truth, so there is a very different principle at work here, and this answer choice is incorrect.
Must Be True—PR. The correct answer choice is (B)
Although this philosopher's assertion is stated in a fairly sophisticated manner, it can be restated in simpler terms:
We shouldn't be opposed to revealing truths we'd rather not acknowledge, or those
that might have negative influence on society. The question stem asks for the answer choice which most closely conforms to this basic principle.
Answer choice (A): Since this choice deals with illegally obtained information, it does not reflect the principle from the stimulus, which deals only with the assertion that we shouldn't be opposed to revealing certain truths.
Answer choice (B): This is the correct answer choice. This choice applies the principle from the stimulus to a medical context. We should not restrict such research even if negative consequences might result.
Answer choice (C): The philosopher makes no such specific exceptions to the general rule that we should not be opposed to unearthing the referenced truths.
Answer choice (D): This answer choice is incorrect for basically the same reason as incorrect answer choice (A) above. Since this choice deals with illegal means of obtaining information, the philosopher's assertion is not applicable in this context.
Answer choice (E): While the stimulus' argument is that we should not be opposed unearthing truths, this choice asserts that in certain contexts we should not have to adhere too closely to the truth, so there is a very different principle at work here, and this answer choice is incorrect.